Difference between revisions of "1971 AHSME Problems/Problem 20"

(Created page with "== Problem == The sum of the squares of the roots of the equation <math>x^2+2hx=3</math> is <math>10</math>. The absolute value of <math>h</math> is equal to <math>\textbf{(...")
 
m (see also, boxed answer, added link)
Line 10: Line 10:
  
 
== Solution ==
 
== Solution ==
We can rewrite the equation as <math>x^2 + 2hx - 3 = 0.</math> By Vieta's Formulas, the sum of the roots is <math>-2h</math> and the product of the roots is <math>-3.</math>
+
We can rewrite the equation as <math>x^2 + 2hx - 3 = 0.</math> By [[Vieta's Formulas]], the sum of the roots is <math>-2h</math> and the product of the roots is <math>-3.</math>
  
 
Let the two roots be <math>r</math> and <math>s.</math> Note that
 
Let the two roots be <math>r</math> and <math>s.</math> Note that
 
<cmath>r^2 + s^2 = (r+s)^2 - 2rs = (-2h)^2 -2(-3)</cmath>
 
<cmath>r^2 + s^2 = (r+s)^2 - 2rs = (-2h)^2 -2(-3)</cmath>
  
Therefore, <math>4h^2 + 6 = 10</math> and <math>h = \pm 1.</math> This doesn't match any of the answer choices, so the answer is <math>\textbf{(E)}.</math>
+
Therefore, <math>4h^2 + 6 = 10</math> and <math>h = \pm 1.</math> This doesn't match any of the answer choices, so the answer is <math>\boxed{\textbf{(E) }\text{None of these}}.</math>
  
 
-edited by coolmath34
 
-edited by coolmath34
 +
 +
== See Also ==
 +
{{AHSME 35p box|year=1971|num-b=19|num-a=21}}
 +
{{MAA Notice}}

Revision as of 09:49, 5 August 2024

Problem

The sum of the squares of the roots of the equation $x^2+2hx=3$ is $10$. The absolute value of $h$ is equal to

$\textbf{(A) }-1\qquad \textbf{(B) }\textstyle\frac{1}{2}\qquad \textbf{(C) }\textstyle\frac{3}{2}\qquad \textbf{(D) }2\qquad \textbf{(E) }\text{None of these}$

Solution

We can rewrite the equation as $x^2 + 2hx - 3 = 0.$ By Vieta's Formulas, the sum of the roots is $-2h$ and the product of the roots is $-3.$

Let the two roots be $r$ and $s.$ Note that \[r^2 + s^2 = (r+s)^2 - 2rs = (-2h)^2 -2(-3)\]

Therefore, $4h^2 + 6 = 10$ and $h = \pm 1.$ This doesn't match any of the answer choices, so the answer is $\boxed{\textbf{(E) }\text{None of these}}.$

-edited by coolmath34

See Also

1971 AHSC (ProblemsAnswer KeyResources)
Preceded by
Problem 19
Followed by
Problem 21
1 2 3 4 5 6 7 8 9 10 11 12 13 14 15 16 17 18 19 20 21 22 23 24 25 26 27 28 29 30 31 32 33 34 35
All AHSME Problems and Solutions

The problems on this page are copyrighted by the Mathematical Association of America's American Mathematics Competitions. AMC logo.png